Holooly Plus Logo

Question 2.27: Assume equation (2.32) can be written in the form ∂z/∂x = f ......

Assume equation (2.32) can be written in the form

F(x,y,z,p,q) = 0,          (2.32)

\frac{\partial z}{\partial x}=f\left(x, y, z, \frac{\partial z}{\partial y}\right)            (2.41)

(for example, in a neighbourhood of a point where it holds  \frac{\partial F}{\partial p} \neq 0.) Then using the method of variation of constants, prove that all the solutions of (2.32) can be obtained from the complete solution.

Step-by-Step
The 'Blue Check Mark' means that this solution was answered by an expert.
Learn more on how do we answer questions.

Hint. Firstly take a = a( x, y) and b = b( x, y) in the complete solution

z = g(x,y,z,a,b).

Then, by the definition of the complete solution, replacing z, a and b with the functions g(x,y,z,a,b), \frac{\partial g}{\partial x}(x, y, z, a, b) \text { and } \frac{\partial g}{\partial y}(x, y, z, a, b) respectively, one gets an identity in (2.41) for arbitrary functions a and b.
Conversely, a function z = g(x, y, z, a, b) is a solution of (2.41) only if

\frac{\partial z}{\partial x}=\frac{\partial g}{\partial x} \text { and } \frac{\partial z}{\partial y}=\frac{\partial g}{\partial y} \text {. }

Show that this will hold only if

\begin{aligned}& \frac{\partial g}{\partial a}(x, y, a, b) \frac{\partial a}{\partial x}+\frac{\partial g}{\partial b}(x, y, a, b) \frac{\partial b}{\partial x}=0 \\& \frac{\partial g}{\partial a}(x, y, a, b) \frac{\partial a}{\partial y}+\frac{\partial g}{\partial b}(x, y, a, b) \frac{\partial b}{\partial y}=0 .\end{aligned}         (2.42)

Putting \frac{\partial g}{\partial a}(x, y, a, b)=0 \text { and } \frac{\partial g}{\partial b}(x, y, a, b)=0, and, if possible, eliminating a and b from equations (2.42), we obtain the singular solution. Note that it depends neither from arbitrary constants nor from arbitrary functions.
Next, assume it holds \frac{\partial(a, b)}{\partial(x, y)}=0, where

\frac{\partial(a, b)}{\partial(x, y)}=\left|\begin{array}{ll}\frac{\partial a}{\partial x} & \frac{\partial a}{\partial y} \\\frac{\partial b}{\partial x} & \frac{\partial b}{\partial y}\end{array}\right|,

and let a and b be some non constant functions. Then the functions a and bare not functionally independent, i.e., there exists a differentiable function w such that b = w(a). This leads to the general solution of (2.41).
Finally, check that any solution of (2.41) reduces to one of the following three ones: singular, complete or general.

Related Answered Questions

Question: 2.36

Verified Answer:

The given condition can be written as a scalar pro...
Question: 2.40

Verified Answer:

One easily finds that p-q+x-y=2 a \quad \te...
Question: 2.39

Verified Answer:

I mode. From the system (2.38) we have \fra...
Question: 2.38

Verified Answer:

If we denote by i, j and k, respectively, the unit...
Question: 2.35

Verified Answer:

From the system (2.38) we have \frac{d x}{\...
Question: 2.32

Verified Answer:

Firstly, we shall introduce a new unknown function...
Question: 2.31

Verified Answer:

One easily finds the complete solution: z = xa + y...
Question: 2.30

Verified Answer:

a) Let z = z(x,y) be the sought after surface and ...
Question: 2.29

Verified Answer:

a) Putting F(x, y, z, p, q)=1+p^2-z q[/late...
Question: 2.28

Verified Answer:

Starting from the complete solution (2.43), we can...